Profit loss

This topic has expert replies
Senior | Next Rank: 100 Posts
Posts: 63
Joined: Tue Apr 06, 2010 10:01 pm
Thanked: 4 times
Followed by:1 members

Profit loss

by prat_agl » Wed Sep 05, 2012 12:52 pm
A photography dealer ordered 60 model X cameras to be sold for $250 each, which represent a 20% markup over the dealer's initial cost for each camera. Of the cameras ordered , 6 were never sold and were returned to manufacturer for a refund of 50% of the dealer's initial cost.what was the dealer's aprrox profit or loss as a percent of dealers initial cost of 60 cameras.
a) 7%loss
b) 13%loss
c) 7% profit
D) 13%profit
e) 15 % profit

Was able to solve it but took me around 4 minutes. Is there any shortcut??

User avatar
MBA Admissions Consultant
Posts: 2279
Joined: Fri Nov 11, 2011 7:51 am
Location: New York
Thanked: 660 times
Followed by:266 members
GMAT Score:770

by Jim@StratusPrep » Wed Sep 05, 2012 2:30 pm
This is a pretty long one to work on. The key is to work quickly between individual prices and group prices. What was your method of solving?
GMAT Answers provides a world class adaptive learning platform.
-- Push button course navigation to simplify planning
-- Daily assignments to fit your exam timeline
-- Organized review that is tailored based on your abiility
-- 1,000s of unique GMAT questions
-- 100s of handwritten 'digital flip books' for OG questions
-- 100% Free Trial and less than $20 per month after.
-- Free GMAT Quantitative Review

Image

Master | Next Rank: 500 Posts
Posts: 101
Joined: Sun Jun 03, 2012 10:10 pm
Thanked: 10 times
Followed by:1 members

by everything's eventual » Wed Sep 05, 2012 6:34 pm
This was my way of doing it. I took just more than 2 minutes but this was due to the final step which required lengthy calculations.

The dealer plans to cell the cameras for USD 250 which is 20% more than dealer's intial cost.

Let the initial cost be "a"

Therefore, a + 0.2a = 250

Solve for "a" , a = 208.

Therefore initial cost = 208 * 60 = $ 12480.

The dealer sold 54 cameras at $ 250 = $ 13500

The dealer sold 6 cameras at $ 104 (50% refund on initial cost) = $ 624

Total selling price = $ 14124.

Profit = (14124 - 12480)/ 12480 = 13 % profit.

GMAT/MBA Expert

User avatar
GMAT Instructor
Posts: 3835
Joined: Fri Apr 02, 2010 10:00 pm
Location: Milpitas, CA
Thanked: 1854 times
Followed by:523 members
GMAT Score:770

by Anurag@Gurome » Wed Sep 05, 2012 7:03 pm
prat_agl wrote:A photography dealer ordered 60 model X cameras to be sold for $250 each, which represent a 20% markup over the dealer's initial cost for each camera. Of the cameras ordered , 6 were never sold and were returned to manufacturer for a refund of 50% of the dealer's initial cost.what was the dealer's aprrox profit or loss as a percent of dealers initial cost of 60 cameras.
a) 7%loss
b) 13%loss
c) 7% profit
D) 13%profit
e) 15 % profit

Was able to solve it but took me around 4 minutes. Is there any shortcut??

Total cost = 60 * ($250/1.2) = 50 * 250
No. of cameras sold = 60 - 6 = 54
Total revenue = 54 * 250
No. of cameras returned = 6
Total refund = 6 * (250/1.2) * 0.5
Therefore, total income = 54 * 250 + 6 * (250/1.2) * 0.5

Hence, the dealer's approximate profit = (54 * 250 + 6 * (250/1.2) * 0.5 - 50 * 250)/(50 * 250) * 100 = [spoiler]13%[/spoiler]

The correct answer is D.
Anurag Mairal, Ph.D., MBA
GMAT Expert, Admissions and Career Guidance
Gurome, Inc.
1-800-566-4043 (USA)

Join Our Facebook Groups
GMAT with Gurome
https://www.facebook.com/groups/272466352793633/
Admissions with Gurome
https://www.facebook.com/groups/461459690536574/
Career Advising with Gurome
https://www.facebook.com/groups/360435787349781/

Senior | Next Rank: 100 Posts
Posts: 63
Joined: Tue Apr 06, 2010 10:01 pm
Thanked: 4 times
Followed by:1 members

by prat_agl » Wed Sep 05, 2012 7:28 pm
Thanks all

User avatar
GMAT Instructor
Posts: 15539
Joined: Tue May 25, 2010 12:04 pm
Location: New York, NY
Thanked: 13060 times
Followed by:1906 members
GMAT Score:790

by GMATGuruNY » Thu Sep 06, 2012 1:55 am
A photography dealer ordered 60 Model X cameras to be sold for $250 each, which represents a 20 percent markup over the dealer's initial cost for each camera. Of the cameras ordered, 6 were never sold and were returned to the manufacturer for a refund of 50 percent of the dealer's initial cost. What was the dealer's approximate profit or loss as a percent of the dealer's initial cost for the 60 cameras?
A.7% loss
B.13% loss
C.7% profit
D.13% profit
E.15% profit
Since the question asks for a PERCENTAGE, ignore the numbers given.
Plug in values that satisfy the given conditions and make the math easy.
The fraction of cameras not sold = 6/60 = 1/10.

Let the number of cameras ordered = 10.
Let the cost per camera = 10.
Total cost = 10*10 = 100.

The number not sold = (1/10)10 = 1.
For this one camera, the refund received = .5(10) = 5.

The number of cameras sold = 9.
With a markup of 20%, the selling price = 12.
Total revenue = 9*12 = 108.

Refund + revenue = 5+108 = 113, a profit of 13%.

The correct answer is D.

By plugging in our own values that satisfy all of the given conditions, the correct answer here can be determined MUCH more quickly.
Essentially, the problem above is no different from the following:
A photography dealer ordered x cameras to be sold for y dollars each, which represents a 20 percent markup over the dealer's initial cost for each camera. Of the cameras ordered, 1/10 were never sold and were returned to the manufacturer for a refund of 50 percent of the dealer's initial cost. What was the dealer's approximate profit or loss as a percent of the dealer's initial cost for the cameras?
Private tutor exclusively for the GMAT and GRE, with over 20 years of experience.
Followed here and elsewhere by over 1900 test-takers.
I have worked with students based in the US, Australia, Taiwan, China, Tajikistan, Kuwait, Saudi Arabia -- a long list of countries.
My students have been admitted to HBS, CBS, Tuck, Yale, Stern, Fuqua -- a long list of top programs.

As a tutor, I don't simply teach you how I would approach problems.
I unlock the best way for YOU to solve problems.

For more information, please email me (Mitch Hunt) at [email protected].
Student Review #1
Student Review #2
Student Review #3

Master | Next Rank: 500 Posts
Posts: 141
Joined: Tue Oct 04, 2011 5:17 am
Thanked: 25 times

by coolhabhi » Thu Sep 06, 2012 12:22 pm
Seeing this problem I just had a doubt whether there will be any on screen calculator that can be used in the exam?? Please let me know.

Master | Next Rank: 500 Posts
Posts: 101
Joined: Sun Jun 03, 2012 10:10 pm
Thanked: 10 times
Followed by:1 members

by everything's eventual » Fri Sep 07, 2012 9:17 am
On screen calculator will be available for the IR section but not for the Quant section.
Last edited by everything's eventual on Fri Sep 07, 2012 9:34 pm, edited 1 time in total.

Senior | Next Rank: 100 Posts
Posts: 58
Joined: Mon Aug 27, 2012 3:09 am
Thanked: 2 times

by akashkumar1987 » Fri Sep 07, 2012 10:42 am
Hi,

According to me answer is E. Below is the explanation

Let x be the C.P

x + 0.2x = 250

Initial Cost of 60 Cameras = 60*250/1.2
Revenue = 60*250
Loss encountered for 6 cameras = 6*250*0.5/1.2

Profit = 60*250 - 60*250/1.2 - 6*250*0.5/1.2

Divide the profit by initial cost of 60 cameras and the answer will be E.

Can u please tell me where i am wrong with respect to you guys as your answer is coming as D

Master | Next Rank: 500 Posts
Posts: 101
Joined: Sun Jun 03, 2012 10:10 pm
Thanked: 10 times
Followed by:1 members

by everything's eventual » Fri Sep 07, 2012 9:44 pm
// Quote//

Profit = 60*250 - 60*250/1.2 - 6*250*0.5/1.2

//Unquote//

This equation should be as follows :

Profit = 54*250 + 6*250*0.5/1.2 - 60*250/1.2

See, out of 60 cameras, the dealer sold 54 at $ 250. He returned the remaining cameras for 50% of the intial cost. So basically he " sold" it back to the manufacturer at half the initial cost. So to calculate the profit, you add these two selling prices and then subtract the initial price from this sum.

User avatar
GMAT Instructor
Posts: 15539
Joined: Tue May 25, 2010 12:04 pm
Location: New York, NY
Thanked: 13060 times
Followed by:1906 members
GMAT Score:790

by GMATGuruNY » Mon Sep 10, 2012 5:59 am
prat_agl wrote:A photography dealer ordered 60 model X cameras to be sold for $250 each, which represent a 20% markup over the dealer's initial cost for each camera. Of the cameras ordered , 6 were never sold and were returned to manufacturer for a refund of 50% of the dealer's initial cost.what was the dealer's aprrox profit or loss as a percent of dealers initial cost of 60 cameras.
a) 7%loss
b) 13%loss
c) 7% profit
D) 13%profit
e) 15 % profit

Was able to solve it but took me around 4 minutes. Is there any shortcut??
One more shortcut:

Of the 60 cameras, 54 earn 20% and 6 lose 50%.
Average earnings = (54*20 - 6*50)/60 = 780/60 = 13.

The correct answer is D.
Private tutor exclusively for the GMAT and GRE, with over 20 years of experience.
Followed here and elsewhere by over 1900 test-takers.
I have worked with students based in the US, Australia, Taiwan, China, Tajikistan, Kuwait, Saudi Arabia -- a long list of countries.
My students have been admitted to HBS, CBS, Tuck, Yale, Stern, Fuqua -- a long list of top programs.

As a tutor, I don't simply teach you how I would approach problems.
I unlock the best way for YOU to solve problems.

For more information, please email me (Mitch Hunt) at [email protected].
Student Review #1
Student Review #2
Student Review #3

Junior | Next Rank: 30 Posts
Posts: 10
Joined: Sat Nov 14, 2009 5:53 pm

by minhphuoc » Sat Nov 24, 2012 10:08 am
Hi,

For those using normal way to solve this problem, I think that we can avoid huge calculation by just replace Cost by letter C and put it in the final formula:
Income: 54*1.2C + 6*0.5C
Cost: 60*C
Then percent =(Income-cost)/cost = (54*1.2C + 6*0.5C - 60*C)/60*C. We can see that we can eliminate C from the fraction --> percent = (54*1.2 + 6*0.5 - 60)/60 = (54*1.2 + 6*0.5)/60 - 1. Here, we can see clearly the weighted average part of the problem or we can simply calculate the fraction to reach the result.